www.vorhilfe.de
- Förderverein -
Der Förderverein.

Gemeinnütziger Verein zur Finanzierung des Projekts Vorhilfe.de.
Hallo Gast!einloggen | registrieren ]
Startseite · Mitglieder · Impressum
Forenbaum
^ Forenbaum
Status VH e.V.
  Status Vereinsforum

Gezeigt werden alle Foren bis zur Tiefe 2

Navigation
 Startseite...
 Suchen
 Impressum
Das Projekt
Server und Internetanbindung werden durch Spenden finanziert.
Organisiert wird das Projekt von unserem Koordinatorenteam.
Hunderte Mitglieder helfen ehrenamtlich in unseren moderierten Foren.
Anbieter der Seite ist der gemeinnützige Verein "Vorhilfe.de e.V.".
Partnerseiten
Weitere Fächer:

Open Source FunktionenplotterFunkyPlot: Kostenloser und quelloffener Funktionenplotter für Linux und andere Betriebssysteme
Forum "Integrationstheorie" - Integral über Gebiet
Integral über Gebiet < Integrationstheorie < Maß/Integrat-Theorie < Analysis < Hochschule < Mathe < Vorhilfe
Ansicht: [ geschachtelt ] | ^ Forum "Integrationstheorie"  | ^^ Alle Foren  | ^ Forenbaum  | Materialien

Integral über Gebiet: Frage (beantwortet)
Status: (Frage) beantwortet Status 
Datum: 12:43 So 20.02.2022
Autor: MasterEd

Aufgabe
<br>
Berechnen Sie das Integral der Funktion 𝑓(𝑥, 𝑦) = 𝑥² + 𝑦² über das unten in rot dargestellte Integrationsgebiet.
Achten Sie bei der Integration auf die korrekte Angabe der Integrationsgrenzen.


<br>
Hier der Link zur Grafik:
[]https://ibb.co/Y0Z14wc

Hallo, ich komme bei dieser Frage irgendwie nicht weiter bzw. finde den Anfang nicht.
Also x²+y²=r² stellt ja einen Kreis um den Ursprung mit Radius r da. Ich vermute, wir sollen die Aufgabe mit einem Doppelintegral lösen - oder bin ich da schon falsch?

Ich bin für jede Hilfe dankbar und habe diese Frage nirgendwo sonst gestellt.

        
Bezug
Integral über Gebiet: Antwort
Status: (Antwort) fertig Status 
Datum: 13:47 So 20.02.2022
Autor: Gonozal_IX

Hiho,

>  Also x²+y²=r² stellt ja einen Kreis um den Ursprung mit Radius r da.

"dar".
Sonst: Korrekt.
Aber der Bereich enthält ja mehr als einen Kreis.
Kannst du den restlichen Bereich ebenfalls beschreiben?

Kleiner Tipp: Einfacher ist, wenn du statt dem Gesamtkreis den Bereich als Halbkreis + Rest beschreibst, dann erhältst du zwei geometrisch einfache Figuren.

Und das Integral über einen Bereich kannst du dann in die Teilbereiche zerlegen.

> Ich vermute, wir sollen die Aufgabe mit einem
> Doppelintegral lösen - oder bin ich da schon falsch?

Das wird darauf hinauslaufen.

Gruß,
Gono

Bezug
                
Bezug
Integral über Gebiet: Frage (beantwortet)
Status: (Frage) beantwortet Status 
Datum: 17:00 So 20.02.2022
Autor: MasterEd

Aufgabe
<br>
 


<br>

Hallo, ich erkenne in dem Bild eigentlich ja nur einen Halbkreis (Fläche Pi/2) und ein Rechteck (Fläche 4*2=8), aber das stimmt ja nicht mit dem Kontrollergebnis überein und wäre gänzlich ohne Integral gerechnet.

Bezug
                        
Bezug
Integral über Gebiet: Antwort
Status: (Antwort) fertig Status 
Datum: 18:35 So 20.02.2022
Autor: chrisno

Du sollst ja auch nicht den Flächeninhalt bestimmen.
Das Doppelintegral ist schon der richtige Ansatz.
Fang mit der recheckigen Fläche an.
Von wo bis wo läuft x, von wo bis wo läuft y?
Dann Doppelintegral über die Funktion ...

Bezug
                                
Bezug
Integral über Gebiet: Frage (beantwortet)
Status: (Frage) beantwortet Status 
Datum: 19:57 So 20.02.2022
Autor: MasterEd

Aufgabe
<br>
 


<br>

Ich kann da nur raten!?

Für den rechteckigen Teil:

[mm]\int_{-1}^{1} {\int_{0}^{4}{f(x,y) dx} dy}[/mm]

Bezug
                                        
Bezug
Integral über Gebiet: Antwort
Status: (Antwort) fertig Status 
Datum: 20:14 So 20.02.2022
Autor: Gonozal_IX

Hiho,

> Ich kann da nur raten!?

Wieso kannst du da nur raten?
Blind geboren und niemand verrät dir, wie das Rechteck aussieht?

> Für den rechteckigen Teil:
>  
> [mm]\int_{-1}^{1} {\int_{0}^{4}{f(x,y) dx} dy}[/mm]

Aha, da sieht doch schon mal gut aus…
Das wäre dann also das Integral über die Rechteckfläche… fehlt jetzt noch das Integral über den Halbkreis.

Welche Grenzen für x und y beschreiben denn nun den Halbkreis?

Gruß,
Gono

Bezug
                                                
Bezug
Integral über Gebiet: Frage (beantwortet)
Status: (Frage) beantwortet Status 
Datum: 20:35 So 20.02.2022
Autor: MasterEd

Aufgabe
<br>
 



<br>
Das "Raten" bezog sich darauf, ob f(x,y) auch für das Rechteck zuständig ist, weil es ja eine Kreisfunktion ist...

Für den Halbkreis dann:
[mm]\int_{-1}^{1}{ \int_{-1}^{0}{f(x,y) dx}dy}[/mm] ?

Bezug
                                                        
Bezug
Integral über Gebiet: Antwort
Status: (Antwort) fertig Status 
Datum: 20:57 So 20.02.2022
Autor: Gonozal_IX

Hiho,

>  Das "Raten" bezog sich darauf, ob f(x,y) auch für das
> Rechteck zuständig ist, weil es ja eine Kreisfunktion
> ist...

*brr* In dem Satz ist alles falsch…
1.) f(x,y) ist in keiner Weise für das gegebene Gebiet "zuständig", sondern du sollst die Funktion f ÜBER das gegebene Gebiet integrieren.
Das Gebiet schränkt also die Werte x,y ein, die du in f einsetzt.

2.) Die Funktion f ist keine "Kreisfunktion".

> Für den Halbkreis dann:
>  [mm]\int_{-1}^{1}{ \int_{-1}^{0}{f(x,y) dx}dy}[/mm] ?

Nein.
Was wäre denn [mm] $\{y \in [-1,1], x \in [-1,0]\}$ [/mm] für ein geometrisches Objekt?
Du sollst nicht raten…

1.) Du hast einen Halbkreis mit welchem Radius?
2.) Welche Gleichung beschreibt einen Kreis mit diesem Radius?
3.) Welche zusätzliche Einschränkung benötigst du, damit aus dem Kreis ein Halbkreis wird?

Gruß,
Gono

Bezug
                                                                
Bezug
Integral über Gebiet: Frage (beantwortet)
Status: (Frage) beantwortet Status 
Datum: 21:15 So 20.02.2022
Autor: MasterEd

Aufgabe
<br>
 


<br>
Was wäre denn [$ [mm] \{y \in [-1,1], x \in [-1,0]\} [/mm] $] für ein geometrisches Objekt?
Ein Rechteck

1.) Du hast einen Halbkreis mit welchem Radius? r=1
2.) Welche Gleichung beschreibt einen Kreis mit diesem Radius? [mm] x^2+y^2=1^2 [/mm]
3.) Welche zusätzliche Einschränkung benötigst du, damit aus dem Kreis ein Halbkreis wird?
Also einen Kreis mit r=1 in Abhängigkeit von x haben wir in der Schule als [mm]f(x)=\sqrt{1-x^2}[/mm] für den oberen Halbkreis bzw als -f(x) für den unteren dargestellt. Aber dann ist es eine Funktion einer Variablen und ich weiß nicht, wie ich das mit dem Doppelintegral und f(x,y) zusammenbringen kann.

Mir blieb leider nichts als vermuten/raten übrig, weil ich es nicht weiß. Das Doppelintegral zum Rechteck habe ich schon ausgerechnet, das kommt mit dem Kontrollergebnis auch hin. Aber mit dem Halbkreis bin ich jetzt komplett raus leider.

Gruß, MasterEd

Bezug
                                                                        
Bezug
Integral über Gebiet: Antwort
Status: (Antwort) fertig Status 
Datum: 21:38 So 20.02.2022
Autor: chrisno

Nun geht es um das, was Du bei dieser Übung lernen sollst.
Zuerst einmal, wie es nicht stimmt:
[mm] $\int_{-1}^{0} \int_{-1}^{+1} [/mm] f(x,y) dy dx$
Da würdest Du über das Rechteck integrieren, in dem der rote Halbkreis liegt.
Die nicht roten Stücke sind zu viel.
Um die nicht mit zu erfassen, müssen die Integralgrenzen richtig gesetzt werden. Da ist nicht mehr so einfach.

Nimm mal an, dass x=-0,5. Wo müsste dann die Integration in y-Richtung beginnen, dass nur im Roten integriert wird? (Vom Rand des Halbkreises unten bis zum Rand des Halbkreises oben)
Nun mach es allgemeiner: für ein x zwischen -1 und 0, bei welchem y beginnt das Integral?
Dies sollst Du hinschreiben.

Es ergibt sich, dass die Integrationsgrenzen für y davon abhängen, bei welchem x man gerade ist.


Bezug
                                                                                
Bezug
Integral über Gebiet: Frage (beantwortet)
Status: (Frage) beantwortet Status 
Datum: 21:57 So 20.02.2022
Autor: MasterEd

Aufgabe
<br>
 


<br>
Hallo crisno,
okay ich sehe das Problem und es tut mir leid aber ich komme einfach nicht drauf.

Bei x=-0,5 habe ich aus einer Skizze die Grenzen y=-0,866 bzw. 0,866 berechnet. Verallgemeinert für x die Grenzen bei [mm] \pm\sqrt{1-x^2}[/mm], aber es kommt nicht hin, wenn ich es eingebe.

Ich komme da nicht von selbst drauf, fürchte ich. Welche Grenzen muss ich denn nehmen?

Liebe Grüße

Bezug
                                                                                        
Bezug
Integral über Gebiet: Antwort
Status: (Antwort) fertig Status 
Datum: 22:23 So 20.02.2022
Autor: chrisno


>  
> Bei x=-0,5 habe ich aus einer Skizze die Grenzen y=-0,866
> bzw. 0,866 berechnet.

[ok]

> Verallgemeinert für x die Grenzen
> bei [mm]\pm\sqrt{1-x^2}[/mm],

[ok]

> aber es kommt nicht hin, wenn ich es
> eingebe.

Da musst Du noch etwas falsch machen.

$ [mm] \int_{-1}^{0} \int_{-\sqrt{1-x^2}}^{+\sqrt{1-x^2}} x^2 [/mm] + [mm] y^2 [/mm] dy dx $
Da mein Taschenrechner nur eindimesional integriert, habe ich zuerst das Integral über y berechnet.
Da stehen noch viele x drin. Auch das Integral lässt sich lösen, aber da bin ich aus dem Training und müsste ein wenig nachschauen. Also habe ich es in den Taschenrechner eingetippt. Die Lösung hat schon HJK geschreiben.

Für mich ist nun Feierabend.



Bezug
                                                                                                
Bezug
Integral über Gebiet: Mitteilung
Status: (Mitteilung) Reaktion unnötig Status 
Datum: 13:48 Mo 21.02.2022
Autor: MasterEd

Ich habe es nun nochmal in den TR eingegeben und nun kommt das Kontrollergebnis raus. Diesen Weg mit der Wurzel kann ich auch nachvollziehen.

Bezug
                                                                                        
Bezug
Integral über Gebiet: Antwort
Status: (Antwort) fertig Status 
Datum: 08:27 Mo 21.02.2022
Autor: fred97


> <br>
>   
>  
> <br>
>  Hallo crisno,
>  okay ich sehe das Problem und es tut mir leid aber ich
> komme einfach nicht drauf.
>  
> Bei x=-0,5 habe ich aus einer Skizze die Grenzen y=-0,866
> bzw. 0,866 berechnet. Verallgemeinert für x die Grenzen
> bei [mm]\pm\sqrt{1-x^2}[/mm], aber es kommt nicht hin, wenn ich es
> eingebe.
>  
> Ich komme da nicht von selbst drauf, fürchte ich. Welche
> Grenzen muss ich denn nehmen?
>  
> Liebe Grüße


Zum Integral über den Halbkreis $K$. Hier bieten sich Polarkoordinaten an:

$x= r [mm] \cos [/mm] t, y=r [mm] \sin [/mm] t$, wobei $r [mm] \in [/mm] [0,1]$ und $t [mm] \in [/mm] [ [mm] \pi [/mm] , 2 [mm] \pi]$. [/mm] Benutze den Transformationssatz !

Wie lautet dann das Integral $ [mm] \int_K (x^2+y^2) [/mm]  d(x,y)$ ?

Bezug
                                                                                                
Bezug
Integral über Gebiet: Mitteilung
Status: (Mitteilung) Reaktion unnötig Status 
Datum: 13:49 Mo 21.02.2022
Autor: MasterEd

Polarkoordinaten verwirren mich jetzt nur noch mehr und Transformationssatz sagt mir gerade nichts. Aber danke trotzdem für die Hilfe. Ich habe es nun mit dem Weg aus der anderen Antwort herausbekommen mit Wurzel als Integrationsgrenzen.

Bezug
                                                                                                        
Bezug
Integral über Gebiet: Mitteilung
Status: (Mitteilung) Reaktion unnötig Status 
Datum: 14:46 Mo 21.02.2022
Autor: fred97


> Polarkoordinaten verwirren mich jetzt nur noch mehr und
> Transformationssatz sagt mir gerade nichts.

Merkwürdig. Seit über 15 Jahren bist Du in diesem Forum in der "Hochschulmathematik" unterwegs .......

> Aber danke
> trotzdem für die Hilfe. Ich habe es nun mit dem Weg aus
> der anderen Antwort herausbekommen mit Wurzel als
> Integrationsgrenzen.


Bezug
        
Bezug
Integral über Gebiet: Mitteilung
Status: (Mitteilung) Reaktion unnötig Status 
Datum: 15:12 So 20.02.2022
Autor: HJKweseleit

Zur Kontrolle: Du solltest [mm] \pi/4 [/mm] + 136/3 [mm] \approx [/mm] 46,12 herausbekommen.

Bezug
        
Bezug
Integral über Gebiet: Antwort
Status: (Antwort) fertig Status 
Datum: 21:16 Di 22.02.2022
Autor: HJKweseleit

Ich schreibe dir jetzt mal eine in meinen Augen völlig einfache Lösung auf.

Wir zerlegen das Gebiet wie beim Lesen in waagerechte Streifen der Dicke dy von links nach rechts und die Streifen von oben nach unten.

[Dateianhang nicht öffentlich]

In der Höhe y ist der x-Wert am linken Rand der roten Zeile [mm] -\wurzel{1-y^2}, [/mm] das gilt auch für negative y-Werte. Der rechte  Rand ist immer 4.

Nun Integrieren wir zunächst den Funktionswert über die Linie:

[mm] \integral_{-\wurzel{1-y^2}}^{4}f(x,y) [/mm] dx

Das machen wir nun über alle Linien der Breite dy von y=-1 bis y=1:

[mm] \integral_{-1}^{1} (\integral_{-\wurzel{1-y^2}}^{4}f(x,y) [/mm] dx)dy und das war's schon.

[mm] \integral_{-1}^{1} (\integral_{-\wurzel{1-y^2}}^{4}f(x,y) [/mm] dx)dy = [mm] \integral_{-1}^{1} (\integral_{-\wurzel{1-y^2}}^{4}x^2 [/mm] + [mm] y^2 [/mm] dx)dy = [mm] \integral_{-1}^{1}(x^3/3 [/mm] + [mm] xy^2 |^4_{-\wurzel{1-y^2}} [/mm] )dy = [mm] \integral_{-1}^{1}(64/3 [/mm] + [mm] 4y^2 [/mm] - [mm] (-\wurzel{1-y^2}^3/3 -\wurzel{1-y^2}y^2))dy [/mm]
=  [mm] \integral_{-1}^{1}(64/3 [/mm] + [mm] 4y^2 +\wurzel{1-y^2}((1-y^2)/3 [/mm]  + [mm] y^2))dy [/mm] = [mm] \integral_{-1}^{1}(64/3 [/mm] + [mm] 4y^2 +\wurzel{1-y^2}(1+2y^2)/3 [/mm] )dy = ... = [mm] \pi/4 [/mm] + 136/3


Dateianhänge:
Anhang Nr. 1 (Typ: JPG) [nicht öffentlich]
Bezug
Ansicht: [ geschachtelt ] | ^ Forum "Integrationstheorie"  | ^^ Alle Foren  | ^ Forenbaum  | Materialien


^ Seitenanfang ^
ev.vorhilfe.de
[ Startseite | Mitglieder | Impressum ]